Is 8^n+47 never a prime? Why? | JBMO Shortlist

  Рет қаралды 36,555

letsthinkcritically

letsthinkcritically

Күн бұрын

Пікірлер: 87
@amitsrivastava1934
@amitsrivastava1934 2 жыл бұрын
An excellent problem with an elegant solution.
@shinoa8605
@shinoa8605 2 жыл бұрын
Thanks a lot, sir, for your work. I am seeing, how I become better everyday. This is the result of your work as well as mine
@gabiturbinca9102
@gabiturbinca9102 2 жыл бұрын
If u use fermats theorem u get that 8^4k=1mod 5 from which 8^(4k+1)+47=8+47=0mod 5, and also 8^4k=2^12k=1mod 13,from which 8^(4k+3)+47=512+47=0mod 13 n even its pretty easy to show that 8^n+47=1+47=0 mod 3
@magicmeatball4013
@magicmeatball4013 2 жыл бұрын
Now prove fermats theorem
@霍金本人
@霍金本人 2 жыл бұрын
@@magicmeatball4013 Google is your best friend
@ignaciobenjamingarridoboba2071
@ignaciobenjamingarridoboba2071 2 жыл бұрын
@@magicmeatball4013 the induction proof is not hard
@zhangmike4852
@zhangmike4852 2 жыл бұрын
the solution is amazingly simple!
@kobethebeefinmathworld953
@kobethebeefinmathworld953 2 жыл бұрын
For Claim #2, I'll suggest using Fermat's little theorem to make the proof shorter and cleaner.
@mahmoudalbahar1641
@mahmoudalbahar1641 2 жыл бұрын
Thank your for great videos... And I suggest that you make video about combinatorial proof for Hexagon identity in pascal triangle.
@goatgamer001
@goatgamer001 2 жыл бұрын
if 8^n is 1 mod 3, then its pretty obvious. if it isnt, it is either divisible by 5 or 13.
@9core
@9core 2 жыл бұрын
yeah this type of problem is almost always easily solvable using that method
@9core
@9core 2 жыл бұрын
yeah this type of problem is almost always easily solvable using that method
@johnnath4137
@johnnath4137 2 жыл бұрын
If n is even let n = 2m. We have 8*n + 47 = 64^m + 47 ≡ 1^m + 47 (mod 3) ≡ 0 (mod 3) ⇒ 3|(8^n + 47) for even n. If n is odd, there are two cases (i) n = 4m + 1 or (ii) n = 4m + 3. Case (i): 8^n + 47 = (4)(256^m) + 47 ≡ (4)(1^m) + 47 (mod 17) ≡ 0 (mod 17) ⇒ 17|(8^n + 47) for odd n of form 4m +1 Case (II): 8^n + 47 = (64)(256)^m + 47 ≡ (1)(1^m) + 47 (mod 3) ≡ 0 (mod 3) ⇒ 3|(8^n + 47) for odd n of form 4n + 3. Thus 8^n + 47 is composite for all n.
@lgooch
@lgooch 2 жыл бұрын
For odd case why didn’t we just do 2m+1?
@johnnath4137
@johnnath4137 2 жыл бұрын
@@lgooch Because there are two types of odd numbers, type 4m + 1 and type34m + 3, and "2n + 1" doesn't discriminate between them. And each type requires a different argument. I'll illustrate further by a nice example. Every prime of the form 4m + 1 can be expressed as the sum of two integer squares but no prime of the form 4m + 3 can be so expressed (a result due to Fermat).
@lgooch
@lgooch 2 жыл бұрын
@@johnnath4137 ah thanks
@leif1075
@leif1075 2 жыл бұрын
@@johnnath4137 but 47 equals 2 mod 3 not zero mod 3?
@johnnath4137
@johnnath4137 2 жыл бұрын
@@leif1075 1 + 47 = 48 ≡ 0 (mod 3).
@richardfredlund3802
@richardfredlund3802 2 жыл бұрын
with the heavy hint re divisibility by 3,5,13 I twigged how to do this by about minute 7.
@bait6652
@bait6652 2 жыл бұрын
Interesint authoer typically likes to play with smaller modulo/negative...but calculated 64^2....instead of just using 64(1,-1,1,-1,1)for %3,5,13
@azai.mp4
@azai.mp4 2 жыл бұрын
Great example of how trying out small values can help you come up with a proof!
@pandagameryt6288
@pandagameryt6288 9 ай бұрын
we know that for a prime p>=7, p=1,-1 (mod 6). if 8^n +47 is a prime, then 8^n +47 = +1,-1 (mod 6). but 8^N+47 = 2^n -1 which should be =1,-1 (mod6). since -1 is is impossible, we take 1. so, 2^n= 2 (mod 6). 2^n-1 =1 mod6. which is never possible for n>=1and the given eq implies p>=55. so, 8^n +47 is never a prime.
@amagilly
@amagilly 2 жыл бұрын
Another masterpiece.
@krishnanadityan2017
@krishnanadityan2017 2 жыл бұрын
When n is even, the given number is divisible by 3.
@SQRTime
@SQRTime 2 жыл бұрын
Hi Krishnan. If you are interested in math competitions, please consider kzbin.info/www/bejne/r6C1dp-gq6x_pa8 and other videos in the Olympiad playlist. Hope you enjoy 😊
@paulchapman8023
@paulchapman8023 2 жыл бұрын
Every power of 4 is two less than a multiple of 3, every power of 64 is also a power of 4, and 47 mod 3 = 2. Yup, that checks out.
@maelmao
@maelmao 2 жыл бұрын
What’s the tablet you use ?
@snehasismaiti342
@snehasismaiti342 2 жыл бұрын
I can't do this problem can anyone solve this "Find the smallest number which has 144 divisors 10 of which are consecutive number."
@kyooz4776
@kyooz4776 2 жыл бұрын
110880=2⁵×3²×5×7×11 notice that because you need to have 10 consecutive divisors at least one of them has to be divisible by 8=2³ and another one by 9=3². You then want to have the smallest primes possible in the factorization of this number. Because 144=2⁴×*3²* two terms will have exponent 3n-1. You can then either have 2³×3^(3-1)×5^(3-1)×7×11 which has 4×3×3×2×2=144 divisors or 2^(6-1)×3^(3-1)×5×7×11 which also has 6×3×2×2×2=144 divisors. The second one is smaller so the answer is 110880
@snehasismaiti342
@snehasismaiti342 2 жыл бұрын
@@kyooz4776 why 3n-1 as exponent of 2 terms
@leif1075
@leif1075 2 жыл бұрын
@@kyooz4776 but the problem as stated doesn't make sense..144 divisors of 10..10 only has 4 divisors 1,2,5 and 10..and what does he mean consecutive numbers?
@kyooz4776
@kyooz4776 2 жыл бұрын
@@leif1075 read again
@kyooz4776
@kyooz4776 2 жыл бұрын
@@snehasismaiti342 because there are 144 divisors, search up number of divisors formula
@dimitardimitrakov2841
@dimitardimitrakov2841 2 жыл бұрын
Proof by contradiciton without mods: 8^n + 64 - 17 = p where p is prime p>=55 and n>2 . Then 8^(n-2)+1 = (p+ 17)/64. left hand side is odd. In the right hand side p is odd because prime => p+17 is even . let p+17 = 2k. Then right hand side becomes k/32 and it must be odd because left hand side is odd. therefor k is odd and let k=2t+1. Finally we have 32*8^(n-2) + 32 = 2t+1 => something even = 2t - 31 which is odd.
@sirak_s_nt
@sirak_s_nt 9 ай бұрын
You cannot claim that k is odd. Even though k/32 is odd, k can be 96 for which k/32 is odd. Use your common sense, if k is odd, how come 32 is going to reduce it to an integer? 32 cannot divide odd numbers.
@ReckonClasses
@ReckonClasses 2 жыл бұрын
Nice question
@ElmanMaharramov
@ElmanMaharramov 2 жыл бұрын
Where can you found this question? Please tell me it
@ibrahimagazade9418
@ibrahimagazade9418 2 жыл бұрын
😅😅
@ElmanMaharramov
@ElmanMaharramov 2 жыл бұрын
@@ibrahimagazade9418 tapdın burdada məni😅
@xeyalehemzeyeva08
@xeyalehemzeyeva08 2 жыл бұрын
😂
@euleri0
@euleri0 2 жыл бұрын
Indigenous!! ⭐
@Szynkaa
@Szynkaa 2 жыл бұрын
tbh awful question, typical checking modulo prime numbers and praying to god for it to work without checking too many of them
@SQRTime
@SQRTime 2 жыл бұрын
Hi Mr Sz. If you are interested in math competitions, please consider kzbin.info/www/bejne/r6C1dp-gq6x_pa8 and other videos in the Olympiad playlist. Hope you enjoy 😊
@muhendisgenc8216
@muhendisgenc8216 2 жыл бұрын
Woooow
@SQRTime
@SQRTime 2 жыл бұрын
Hi. If you are interested in math competitions, please consider kzbin.info/www/bejne/r6C1dp-gq6x_pa8 and other videos in the Olympiad playlist. Hope you enjoy 😊
@ElieBensaid
@ElieBensaid 4 ай бұрын
14 mins just to study that number in 3 different mods, kind of a waste of time
@aftorucova9980
@aftorucova9980 Жыл бұрын
Does anyone have JBMO 2022 shortlist?
@wookong1723
@wookong1723 2 жыл бұрын
loveeeeeeee
@amirhosseinmohajerinejad1393
@amirhosseinmohajerinejad1393 2 жыл бұрын
Nice
@atpugnes
@atpugnes 2 жыл бұрын
I am little curious. Whenever you multiply, you start from the most significant digit. How is it so? Normally we would start from units place.
@sdspivey
@sdspivey 2 жыл бұрын
He isn't multiplying, he's just copying the result he already calculated.
@bornfromstardust1526
@bornfromstardust1526 2 жыл бұрын
The method is called, Calculator Inclusion method.
@poproporpo
@poproporpo 2 жыл бұрын
Yes, even though he most likely copied computations that he had previously completed, there are mental math calculation methods that allow you to calculate from the most significant digit to the least significant digit. As you calculate, you would have to go back and change the result that you have already completed if you need to carry digits. Many mental arithmetic competitors prefer this method iirc.
@EternalLoveAnkh
@EternalLoveAnkh 2 жыл бұрын
I don't mean to criticize but wouldn't it be easier to ask if it is ever prime? RJ
@Sp1tz1fy
@Sp1tz1fy 2 жыл бұрын
For photo math, it turns out as a exponential function
@Ming-pt6wt
@Ming-pt6wt Жыл бұрын
photo math can't do number theory questions.
@王剛-m7n
@王剛-m7n 2 жыл бұрын
3.5.13 mod cover 2n,4n+1, 4n+3
@JinhaoPan-np7zy
@JinhaoPan-np7zy Ай бұрын
写过
@xeyalehemzeyeva08
@xeyalehemzeyeva08 2 жыл бұрын
Would you please tell me where to find it?
@ElmanMaharramov
@ElmanMaharramov 2 жыл бұрын
Məndə soruşmuşam hələ cavab vermiyib
@xeyalehemzeyeva08
@xeyalehemzeyeva08 2 жыл бұрын
@@ElmanMaharramov deyəsən vermiyəcək..
@joaozin003
@joaozin003 2 жыл бұрын
n=-infinity
@hijeffhere
@hijeffhere 2 жыл бұрын
What app are you using to write these? Thank you!
@browhat6935
@browhat6935 2 жыл бұрын
Try latex codecogs
@typo691
@typo691 2 жыл бұрын
it's onenote, he wrote in a community post once
@hijeffhere
@hijeffhere 2 жыл бұрын
Thank you, people!
@donaldcoombs79
@donaldcoombs79 2 жыл бұрын
donnkey kong mario cart
@haleshs66
@haleshs66 2 жыл бұрын
13:06 How can 4096^k always have a remainder of 1 when divided by 13 ???
@crazycat1503
@crazycat1503 2 жыл бұрын
When you multiply numbers, their remainders also multiplies
@mcwulf25
@mcwulf25 2 жыл бұрын
(4095+1)*(4085+1)*.... where 13|4095. Multiply out the brackets and each term apart from the 1*1*1*... is divisible by 13.
@hhgygy
@hhgygy 2 жыл бұрын
@@mcwulf25 Thanks I also missed that step why 64k would always yield a remainder of 1
@leif1075
@leif1075 2 жыл бұрын
@@mcwulf25 where did you get 4095 from? And why out the extra 1 there?
@mcwulf25
@mcwulf25 2 жыл бұрын
@@leif1075 All I am doing is replacing 4096 with 4095+1. That 4085 should be 4095.
@iDovahkiin
@iDovahkiin 2 жыл бұрын
You mean: "Is 8^n+47 is always prime"? Idk if I'm wrong but aren't a prime number is a number that is only dividable by itself and 1? So 8+47=55 which is a prime number??
@ervinforever9953
@ervinforever9953 2 жыл бұрын
The word **only** is the key. 55 is divisible with 5 and 11 as well.
@iDovahkiin
@iDovahkiin 2 жыл бұрын
@@ervinforever9953 how did i not realize 🤦
@leif1075
@leif1075 2 жыл бұрын
Isn't there a way tonsolve without using mod??
@adb012
@adb012 2 жыл бұрын
Yes. The same way. Just use division and remainder. For example, instead of saying "note that 64^k is 1 (mod 3)" say "note that if you divide 64^k by 3 you get a remainder of 1, because 63 is multiple of 3".
@adb012
@adb012 2 жыл бұрын
@@adityaekbote8498 ... Given that the question is asking if something is prime, and prime means that it is only divisible by itself and 1, and divisible means that the reminder of the division is zero, I doubt that it can be proved without using this concept. You can disguise it as "being in the time table of...", or doing repeated subtractions instead of dividing, or stuff like that. But at the end of the day, to show that something is NEVER prime you need to show that is ALWAYS divisible by something other than itself and 1.
@dimitardimitrakov2841
@dimitardimitrakov2841 2 жыл бұрын
yes, i wrote it in the comments, if you use odd vs even.
@mr.kaiden7159
@mr.kaiden7159 8 ай бұрын
If u don't know fermat's theorm U can use (a+b)^n =b^n MOD a we divide into four case for n=4k+i,for k=0,1,2,3,... and i=0,1,2,3 Now, we know that's number is odd ans 47=-1 MOD 3 47=2 MOD 5 47=-2 MOD 7 47=2 MOD 9 47= 3 MOD 11 47= -5 MOD 13 For case i=0,2 8^(4k+i)=(9-1)^(4k+i) Because i is even and 3|9 then 8^(4k+i)=1 MOD 3 For i=1 so 4k+1 is odd 8^(4k+1)=(10-2)^(4k+1)=-2 MOD 5 For i=3 so 4k +3 is odd 8^(4k+3)=(13-5)^(4k+3)=(-5)^(4k+3) MOD 13=(-5)((26-1)^(2k+1)) MOD 13=(-5)(-1) MOD 13=5 MOD 13 So 8^n+47 always can divided by 3,5,or 13
@yoav613
@yoav613 2 жыл бұрын
Why are you negative? Ask is 8^n+47 always composite,and be positive!
@ReckonClasses
@ReckonClasses 2 жыл бұрын
Nice question
Find the Missing Digits. NO CALCULATOR!!
10:47
letsthinkcritically
Рет қаралды 36 М.
A Beautiful Exponential Equation
12:26
letsthinkcritically
Рет қаралды 19 М.
Ozoda - Lada (Official Music Video)
06:07
Ozoda
Рет қаралды 7 МЛН
iPhone or Chocolate??
00:16
Hungry FAM
Рет қаралды 34 МЛН
АЗАРТНИК 4 |СЕЗОН 2 Серия
31:45
Inter Production
Рет қаралды 1,1 МЛН
Как мы играем в игры 😂
00:20
МЯТНАЯ ФАНТА
Рет қаралды 3 МЛН
Why is 2^n + 3^n never a perfect cube? | JBMO Shortlist
8:59
letsthinkcritically
Рет қаралды 18 М.
This Result Keeps Me Up At Night
8:53
BriTheMathGuy
Рет қаралды 1 МЛН
Crisis in the Foundation of Mathematics | Infinite Series
12:40
PBS Infinite Series
Рет қаралды 965 М.
When is p^2-p+1 a Cube? | Balkan MO 2005
8:56
letsthinkcritically
Рет қаралды 13 М.
A Functional Equation That Only Requires the Simplest Knowledge
10:25
letsthinkcritically
Рет қаралды 25 М.
What's special about 277777788888899? - Numberphile
14:24
Numberphile
Рет қаралды 2,2 МЛН
In 2003 We Discovered a New Way to Generate Primes
22:17
Eric Rowland
Рет қаралды 405 М.
Kramnik Went Nuts | Accuses the Indian Olympiad Team
12:19
ChessPulse Hub
Рет қаралды 58 М.
Ozoda - Lada (Official Music Video)
06:07
Ozoda
Рет қаралды 7 МЛН